Đến nội dung

Hình ảnh

Marathon Tổ hợp và rời rạc VMF

- - - - - tổ hợp rời rạc marathon

  • Please log in to reply
Chủ đề này có 65 trả lời

#1
bangbang1412

bangbang1412

    Độc cô cầu bại

  • Phó Quản lý Toán Cao cấp
  • 1667 Bài viết

Được sự góp ý và đồng ý , trao đổi cùng các bạn Ego , baopbc , No Moniker mình xin phép mở topic marathon về tổ hợp và rời rạc đồng thời khi đó các topic về bất đẳng thức , số học , hình học đã khá sôi nổi .

Các chủ đề thảo luận trong topic:

Các chuyên đề : 

+ Tập hợp

+ Ánh xạ 

+ Chỉnh hợp tổ hợp 

+ Quy tắc cộng và nhân

+ PP truy hồi 

+ PP hàm sinh

+ PP ánh xạ

+ PP quỹ đạo 

+ PP đa thức , số phức 

+ Nguyên tắc Dirichle

+ Nguyên lý cực hạn

+ Các bài toán tô màu 

+ Bất biến , đơn biến

+ Quy nạp 

+ Bất biến đơn biến 

+ Lý thuyết đồ thị và hình học tổ hợp 

+ Phương pháp khác ( nêu rõ nếu nó không thuộc các phương pháp trên ) 

Các dạng toán : 

+ Đếm 

+ Đẳng thức tổ hợp 

+ Cực trị tổ hợp 

+ Đặc tính tổ hợp 

+ Một số dạng khác 

Nội dung cuộc thi là chúng ta sẽ lần lượt đăng các bài toán , giải đúng bạn sẽ được thêm 1 điểm và có quyền đề xuất bài toán mới . Lưu ý chỉ người giải mới được đề xuất , có thể nhờ người khác đề xuất hộ . 

Các quy định phải tuân thủ : 

 1. Chỉ đăng các bài toán về tổ hợp rời rạc

 2. Không được giải bài toán do chính mình đề xuất , không được đăng bài toán của các cuộc thi vẫn chưa kết thúc ở các tạp chí , diễn đàn , ....

 3. Ghi rõ nguồn bài toán nếu có . ( nếu tự nghĩ bạn có thể ghi tên mình ) 

 4. Không spam , lời giải rõ ràng , không vắn tắt làm khó hiểu người đọc . 

 5. Mỗi bài đăng của bạn sẽ theo form sau mỗi khi bạn giải xong bài toán thứ $n$ 

    Lời giải bài toán n : 

   Bài toán n+1 ( nguồn ) : Tiếp đó là bài mà bạn đề xuất

6. Không đăng các bài toán mở , các giả thuyết , ...

7. Nếu một bài toán trong $4$ ngày không được giải chúng ta sẽ đăng bài toán khác và đánh dấu lại bài toán đó . Nếu sau đó ai giải được sẽ được cộng hai điểm . Chúng ta có thể đăng lời giải bài toán nào đó đã có người giải rồi miễn là không trùng với lời giải cũ nhưng điểm chỉ dành cho người nhanh và đúng đầu tiên . 

8. Các bài toán đăng lên độ khó nhất định , có thể không quá khó nhưng yêu cầu tư duy và suy nghĩ .

9. Lời giải không rõ ràng , cụt ngủn , lan man sẽ được cộng $0,5$ thôi , bài viết spam sẽ bị xóa

Mong các ĐHV lưu ý và góp sức 

10. Lưu ý nếu một bài toán khó các bạn có ý tưởng cũng có thể chia sẻ để mọi người cùng nhau giải .


Bài viết đã được chỉnh sửa nội dung bởi bangbang1412: 26-05-2016 - 19:54

$$[\Psi_f(\mathbb{1}_{X_{\eta}}) ] = \sum_{\varnothing \neq J} (-1)^{\left|J \right|-1} [\mathrm{M}_{X_{\sigma},c}^{\vee}(\widetilde{D}_J^{\circ} \times_k \mathbf{G}_{m,k}^{\left|J \right|-1})] \in K_0(\mathbf{SH}_{\mathfrak{M},ct}(X_{\sigma})).$$


#2
bangbang1412

bangbang1412

    Độc cô cầu bại

  • Phó Quản lý Toán Cao cấp
  • 1667 Bài viết

Bài toán 1 : ( VMO 2015 )  Cho số nguyên dương $k$ . Tìm số các số tự nhiên $n$ không vượt quá $10^{k}$ thỏa mãn : 

$i)$ $n$ là bội của $3$

$ii)$ Các chữ số trong biểu diễn thập phân của $n$ là $2,0,1,5$

Bắt đầu bằng một bài đếm quen thuộc ^_^ hi vọng các bạn không chê 


$$[\Psi_f(\mathbb{1}_{X_{\eta}}) ] = \sum_{\varnothing \neq J} (-1)^{\left|J \right|-1} [\mathrm{M}_{X_{\sigma},c}^{\vee}(\widetilde{D}_J^{\circ} \times_k \mathbf{G}_{m,k}^{\left|J \right|-1})] \in K_0(\mathbf{SH}_{\mathfrak{M},ct}(X_{\sigma})).$$


#3
I Love MC

I Love MC

    Đại úy

  • Thành viên nổi bật 2016
  • 1861 Bài viết

BÀI TOÁN HIỆN TẠI 

Bài 11 : Trên $1$ đoạn thẳng từ trái sang phải có đặt $21$ hộp gỗ, trong mỗi hộp gỗ có đặt $1$ vài hòn bi. Biết rằng số hòn bi trong mỗi hộp gỗ nhận $1$ trong các giá trị từ $1$ đến $21$ và không có $2$ hộp nào chứa cùng số bi. Mỗi bước ta sẽ chọn $2$ hộp gỗ bất kì và nếu $1$ hộp chứa nhiều bi hơn hộp kia thì ta sẽ chuyển từ hộp nhiều bi hơn sang hộp ít bi hơn $1$ số bi đúng bằng số bi mà hộp ít bi hơn có trước khi chuyển bi. Hỏi có thể luôn chuyển được bi sao cho sau các bước chuyển đó,$ 21$ hộp có số bi nhận $1$ trong các giá trị từ $1$ đến $21$, không có $2$ hộp nào có cùng số bi và số bi của các hộp từ trái sang phải sắp xếp theo thứ tự tăng dần?

Bài toán còn đọng :  Anne và Alice ở $2$ phòng khác nhau. Alice đặt $13$ quân bài từ Át đến K vào $14$ ô trống nằm trên một đường tròn và có $1$ ô không đặt lá bài nào, Anne không thể thấy được Alice sắp xếp thế nào. Mỗi lượt Anne sẽ gọi tên $1$ lá bài. Nếu lá bài đó nằm cạnh ô trống thì Alice sẽ dịch chuyển lá bài đó về phía ô trống, còn nếu không thì không làm gì cả.Sau $1$ số bước thì Anne sẽ dùng hỏi bài.

a/ Anne có một cách nào đó để chắc chắn sau khi dừng hỏi bài thì tất cả các quân bài không còn nằm ở vị trí ban đầu của nó nữa không?

b/  Anne có một cách nào đó để sau chắc chắn sau khi dừng hỏi bài thì quân Át không nằm cạnh ô trống không?


Bài viết đã được chỉnh sửa nội dung bởi bangbang1412: 29-05-2016 - 01:09


#4
No Moniker

No Moniker

    Binh nhất

  • Thành viên mới
  • 26 Bài viết

Bài toán 1 : ( VMO 2015 )  Cho số nguyên dương $k$ . Tìm số các số tự nhiên $n$ không vượt quá $10^{k}$ thỏa mãn : 

$i)$ $n$ là bội của $3$

$ii)$ Các chữ số trong biểu diễn thập phân của $n$ là $2,0,1,5$

Bắt đầu bằng một bài đếm quen thuộc ^_^ hi vọng các bạn không chê 

Bài này chắc khá quen thuộc rồi :D

Xét đa thức $P(x)=(x^5+x^2+x+1)^k$

Dễ thấy tổng các hệ số của đa thức cũng bằng số các bộ $\left ( a_1,a_2,..,a_k \right )\in \begin{Bmatrix} 2;0;1;5\end{Bmatrix}^k$ bằng $4^k.$

hơn nữa số bộ số này chia hết cho 3 cũng bằng tổng hệ số $x^{3k}$ trong đa thức.

Theo định lí Ruf: $S=S=\frac{1}{3}[P(1)+P(\epsilon )+P(\epsilon^2 )]$

Gọi $\epsilon$ là nghiệm của phương trình $x^2+x+1=0$ khi đó phương trình có nghiệm phức $\epsilon_t=cos\frac{2t\pi }{3}+isin\frac{2t\pi }{3}$

Từ đây ta có $\epsilon^3=1$ do đó $1+\epsilon+\epsilon^{2k}=0$ với $k$ không chia hết cho 3 và bằng 3 với $k$ chia hết cho 3

Dễ dàng tính được  $P(1)=4^k,P(\epsilon )=\epsilon^{2k},P(\epsilon^2)=\epsilon^{4k}$

Từ đây suy ra $\left\{\begin{matrix} S=\frac{4^K-1}{3},K\neq 3m\\ S=\frac{4^K+2}{3},K=3m \end{matrix}\right.,m \in \mathbb{Z}$
__________________________
Bài toán đề xuất: 
$\boxed{\text{Bài toán 2}}$ : Trong một cuộc họp có $12k$ người tham gia, mỗi người bắt tay với đúng $3k+6$ người khác. Biết rằng với bất kỳ một cách chọn cặp $2$ người ta có số người bắt tay với cả hai là như nhau. Hỏi có bao nhiêu người tham gia cuộc họp đó?
$$\begin{array}{| l | l |} \hline NoMoniker & 1\\ \hline \end{array}$$


Bài viết đã được chỉnh sửa nội dung bởi baopbc: 26-05-2016 - 21:10

I AM UNNAMED


#5
nhungvienkimcuong

nhungvienkimcuong

    Thiếu úy

  • Hiệp sỹ
  • 669 Bài viết

Bài toán đề xuất: 

$\boxed{\text{Bài toán 2}}$ : Trong một cuộc họp có $12k$ người tham gia, mỗi người bắt tay với đúng $3k+6$ người khác. Biết rằng với bất kỳ một cách chọn cặp $2$ người ta có số người bắt tay với cả hai là như nhau. Hỏi có bao nhiêu người tham gia cuộc họp đó?

Đề nó mâu thuẫn sao ấy nhỉ?   :wacko:


Bài viết đã được chỉnh sửa nội dung bởi baopbc: 27-05-2016 - 09:14

  • Ego yêu thích

Đừng khóc vì chuyện đã kết thúc hãy cười vì chuyện đã xảy ra ~O) 
Thật kì lạ anh không thể nhớ đến tên mình mà chỉ nhớ đến tên em :wub:
Chúa tạo ra vũ trụ của con người còn em tạo ra vũ trụ của anh :ukliam2:


#6
bangbang1412

bangbang1412

    Độc cô cầu bại

  • Phó Quản lý Toán Cao cấp
  • 1667 Bài viết

Lời giải bài 2: Bài này counting in two ways 

Đếm bộ ba $(A,B,C)$ các người , trong đó $A$ bắt tay với $B,C$ nhưng $B,C$ lại không bắt tay với nhau 

Cách $1$: Có $12k$ cách chọn $A$ , sau đó có $3k+6$ người bắt tay với $B$ nên có $3k+6$ cách , riêng với $C$ có $3k+5-t$ người bắt tay với $A$ nhưng không bắt tay với $B$ .Vậy có $12k(3k+6)(3k+5-t)$ ( $t$ là số người bắt tay chung với cả hai người ) 

Cách $2$: Có $12k$ cách chọn $B$ , và $12k-1-(3k+6)=9k-7$ cách chọn $C$ , riêng $A$ có thể chọn $t$ cách 

Vậy có $(3k+6)(3k+5-t)=t(9k-7)$ ,để $t$ nguyên thì $k=3$ dẫn đến có $36$ người 

Tiếp tục nào

Bài toán 3 : Giả sử các tập $A_{1},...A_{k}$ là tập con của $[10]$ với $[n]$ là tập $[n]$ số tự nhiên đầu tiên ( khác $0$)

Thỏa mãn các điều kiẹn

$i)$ Với mọi $i$ thì $|A_{i}|=5$ 

$ii)$ Giao hai tập bất kì không quá $2$

Tìm max của $k$

$$\begin{array}{| l | l |} \hline NoMoniker & 1\\ \hline bangbang1412 & 1\\ \hline \end{array}$$


Bài viết đã được chỉnh sửa nội dung bởi baopbc: 26-05-2016 - 21:11

$$[\Psi_f(\mathbb{1}_{X_{\eta}}) ] = \sum_{\varnothing \neq J} (-1)^{\left|J \right|-1} [\mathrm{M}_{X_{\sigma},c}^{\vee}(\widetilde{D}_J^{\circ} \times_k \mathbf{G}_{m,k}^{\left|J \right|-1})] \in K_0(\mathbf{SH}_{\mathfrak{M},ct}(X_{\sigma})).$$


#7
nhungvienkimcuong

nhungvienkimcuong

    Thiếu úy

  • Hiệp sỹ
  • 669 Bài viết

Bài toán 3 : Giả sử các tập $A_{1},...A_{k}$ là tập con của $[10]$ với $[n]$ là tập $[n]$ số tự nhiên đầu tiên ( khác $0$)

Thỏa mãn các điều kiẹn

$i)$ Với mọi $i$ thì $|A_{i}|=5$ 

$ii)$ Giao hai tập bất kì không quá $2$

Tìm max của $k$

Bài này thì nhiều cách rồi nhỉ ,lập bảng,chặn $\text{plotkin}$,... và sau đây là 1 cách

dễ lập bảng với điều kiện $k=6$ do đó $k\ge 6$

Spoiler

xét với $k\ge 7$

ta gọi $d(i)$ là số tập chưa phần tử $i$ từ đó dễ thấy

$\sum_{i=1}^{10}d_i=\sum_{i=1}^{k}\left | A_i \right |=5k\geq 35$

$\Rightarrow \exists i_0:d_{i_0}\ge 4$ WLOG đó là các tập $A_1,A_2,A_3,A_4$

$\Rightarrow \left | A_1\cap A_2\cap A_3\cap A_4 \right |=1\Rightarrow \left | A_i\cap A_j\cap A_k \right |\geq \left | A_1\cap A_2\cap A_3\cap A_4 \right |=1,\forall 1\le i,j,k\le 4$

do đó 

$$10=\left | A_1\cup ...A_k \right |\geq \left | A_1\cup A_2\cup A_3\cup A_4 \right |=\sum \left | A_i \right |-\sum \left | A_i\cap A_j \right |+\sum \left | A_i\cap A_j\cap A_k \right |-\left | A_1\cap A_2\cap A_3\cap A_4 \right |\ge 4.5-C_4^2.2+(C_4^3-1).1=11$$

điều trên vô lí do đó $k=6$ là giá trị cần tìm

--------------------------------------------------------------------

$\boxed{\text{Bài}\ 4}$(thầy Hà Huy Khoái)

Xét dãy số như sau:$x_1=1$ với $i\ge 2$ thì $x_i$ nhận được từ $x_{i-1}$ bằng cách đổi (trong cách viết số $x_{i-1}$) $1\rightarrow 01,0\rightarrow 1$.Làm như vậy ta nhận được dãy $1,01,101,01101,..$.Trong dãy trên gọi $a_n$ là vị trí của chữ số $1$ thứ $n$,$b_n$ là vị trí của chữ số $0$ thứ $n$ $\left ( a_1=1,b_1=2,a_2=3,a_3=4,b_2=5,a_4=6,b_3=7,... \right )$

Tìm công thức xác định $a_n,b_n$


Bài viết đã được chỉnh sửa nội dung bởi nhungvienkimcuong: 29-05-2016 - 05:06

Đừng khóc vì chuyện đã kết thúc hãy cười vì chuyện đã xảy ra ~O) 
Thật kì lạ anh không thể nhớ đến tên mình mà chỉ nhớ đến tên em :wub:
Chúa tạo ra vũ trụ của con người còn em tạo ra vũ trụ của anh :ukliam2:


#8
bangbang1412

bangbang1412

    Độc cô cầu bại

  • Phó Quản lý Toán Cao cấp
  • 1667 Bài viết

Lời giải bài 4:

Gọi $k_{n}$ là số số $0$ đứng trước số $1$ thứ $n$ , hiển nhiên $a_{n}=n+k_{n}$

Chữ số $0$ thứ $n$ được chuyển từ chữ số $1$ thứ $n$ , chứ số $1 \to 01 \to 101$ . Trước chữ số $1$ thứ $n$ có $k_{n}$ chữ số $0$ . Từ đó $b_{n}=k_{n}+2n$ nên $b_{n}-a_{n}=n$

Định lý $Beatty$ , hai số $\alpha , \beta$ thỏa $\frac{1}{\alpha}+\frac{1}{\beta}=1$ , thế thì đặt $S(a)=([na]|n \in Z)$ ta có $S(\alpha)$ hợp với $S(\beta)$ vét hết tập các số nguyên dương $N^{*}$

Đinh lý này có liên quan đến rất nhiều bài toán dạng dạng này , các bạn nên tìm hiểu chứng minh

Để ý rằng hai dãy $(a_{n})$ và $(b_{n})$ đều tăng , mỗi số nguyên dương xuất hiện một lần trong dãy , ta viết lại như sau

$i$ ta có $a_{1}=1$

$ii b_{n}-a_{n}=n$

$iii  a_{n}$ là số nhỏ nhất không thuộc tập $(a_{1},...a_{n-1},b_{1},...b_{n-1})$

Từ đó suy ra mỗi số nguyên dương xuất hiện đúng $1$ lần ở $1$ trong hai dãy $(a_{n})$ hoặc $(b_{n})$ và hai dãy này xác định duy nhất

Theo cái định lý Beatty kia thì ta sẽ tìm $a_{n},b_{n}$ dưới dạng $a_{n}=[nu],b_{n}=[nv]$ , ta cứ tìm như thế này nếu nó thỏa mãn thì nó đúng vì nó xác định duy nhất mà .

Trong đó $\frac{1}{u}+\frac{1}{v}=1$

Lại có $n=n+[nu]-[nu]=[n(u+1)]-[nu]$ cần có $u+1=v$

Vậy tức là $u$ thỏa mãn $\frac{1}{u}+\frac{1}{u+1}=1$

Từ đó tìm được $(a_{n},b_{n})=([\frac{1+\sqrt{5}}{2}n],[\frac{1+3\sqrt{5}}{2}n])$

Bài 5: Cho số nguyên dương $n>1$ và tập $(1,2,...,n)=S$ . Một tập con của $S$ là cấp số cộng cực đại nếu nó là cấp số cộng và không thể bổ sung thêm số nào để thành cấp số cộng cực đại nữa. Tìm số csc cực đại ? 


Bài viết đã được chỉnh sửa nội dung bởi bangbang1412: 26-05-2016 - 23:54

$$[\Psi_f(\mathbb{1}_{X_{\eta}}) ] = \sum_{\varnothing \neq J} (-1)^{\left|J \right|-1} [\mathrm{M}_{X_{\sigma},c}^{\vee}(\widetilde{D}_J^{\circ} \times_k \mathbf{G}_{m,k}^{\left|J \right|-1})] \in K_0(\mathbf{SH}_{\mathfrak{M},ct}(X_{\sigma})).$$


#9
Long Phi

Long Phi

    Binh nhất

  • Thành viên
  • 26 Bài viết

Không biết có phải hiểu nhầm đề không nữa

Đầu tiên đếm số các công bội, từ $1$ đến $\left \lfloor \frac{n-1}{2} \right \rfloor$

Với mỗi công bội $i$,ta đếm số điểm bắt đầu cấp số cộng, từ $1$ đến $j$ sao cho $j+2i \leqslant n$ hay $j\leqslant n-2i$ và $j\leqslant i$.

Như vậy số csc là $\sum_{1}^{\left \lfloor \frac{n}{3} \right \rfloor}i +\sum_{\left \lfloor \frac{n}{3} \right \rfloor +1}^{\left \lfloor \frac{n-1}{2} \right \rfloor}(n-2i)$


Bài viết đã được chỉnh sửa nội dung bởi Long Phi: 27-05-2016 - 09:54


#10
bangbang1412

bangbang1412

    Độc cô cầu bại

  • Phó Quản lý Toán Cao cấp
  • 1667 Bài viết

Không biết có phải hiểu nhầm đề không nữa

Bạn giải rõ ràng ra đi , đáp số và cách giải 


Bài viết đã được chỉnh sửa nội dung bởi bangbang1412: 27-05-2016 - 13:08

$$[\Psi_f(\mathbb{1}_{X_{\eta}}) ] = \sum_{\varnothing \neq J} (-1)^{\left|J \right|-1} [\mathrm{M}_{X_{\sigma},c}^{\vee}(\widetilde{D}_J^{\circ} \times_k \mathbf{G}_{m,k}^{\left|J \right|-1})] \in K_0(\mathbf{SH}_{\mathfrak{M},ct}(X_{\sigma})).$$


#11
JUV

JUV

    Trung sĩ

  • Điều hành viên OLYMPIC
  • 138 Bài viết

Lời giải bài 5:

 

Chọn ra 2 số thuộc $a,b$ thuộc $S$ mà $a,b$ phân biệt mà $2a\leq n< 2b$. Xét 1 cặp $(a;b)$ như thế, ta thấy rằng chỉ có 1 cấp số cộng cực đại duy nhất chứa 2 số $a$ và $b$ đó mà không chứa bất kì số nào nằm giữa 2 số đó. Tương tự, 2 cấp số cộng cực đại công sai khác 0 mà có chứa 2 số (a;b) thoả mãn  $2a< n\leq 2b$ và không chứa bất kì số nào nằm giữa 2 số đó thì sẽ trùng nhau. Vì vậy có thể thiết lập 1 song ánh giữa tập các cặp $(a;b)$ vào tập các csc cực đại công sai khác 0.Mà có thể tạo được $n$ csc cực đại công sai =0 nên số các csc cực đại bằng số cách chọn cặp $(a;b)$ cộng thêm $n$ và sẽ bằng : $(\frac{n}{2})(\frac{n}{2})+n$ nếu $n$ chẵn và bằng $(\frac{n-1}{2}+1)(\frac{n-1}{2})+n$ nếu $n$ lẻ.

Bài 6:

Cho $n$ hòn sỏi được đặt vào $n$ điểm trên 1 hình tròn. Mỗi lần ta dịch chuyển 2 hòn sỏi , mỗi hòn sỏi được dịch chuyển vào 1 trong 2 điểm mà nằm kề với hòn sỏi đó nhưng chiều dịch chuyển của 2 hòn sỏi đó ngược nhau (có 2 hướng dịch chuyển là cùng và ngược chiều kim đồng hồ). Ta muốn dịch chuyển tất cả các hòn sỏi vào 1 điểm. CMR:

a/ Ta có thể đạt được mục đích nếu $n$ lẻ

b/ Ta không thể đạt được mục đích nếu $n$ chẵn


Bài viết đã được chỉnh sửa nội dung bởi bangbang1412: 27-05-2016 - 13:08


#12
Long Phi

Long Phi

    Binh nhất

  • Thành viên
  • 26 Bài viết

Lời giải bài 5:

Chọn ra 2 số thuộc $a,b$ thuộc $S$ mà $a,b$ phân biệt mà $2a\leq n< 2b$. Xét 1 cặp $(a;b)$ như thế, ta thấy rằng chỉ có 1 cấp số cộng cực đại duy nhất chứa 2 số $a$ và $b$ đó mà không chứa bất kì số nào nằm giữa 2 số đó. Tương tự, 2 cấp số cộng cực đại mà có chứa 2 số (a;b) thoả mãn  $2a< n\leq 2b$ và không chứa bất kì số nào nằm giữa 2 số đó thì sẽ trùng nhau. Vì vậy có thể thiết lập 1 song ánh giữa tập các cặp $(a;b)$ vào tập các csc cực đại. Vì vậy số các csc cực đại bằng số cách chọn cặp $(a;b)$ và sẽ bằng : $(\frac{n}{2})(\frac{n}{2})$ nếu $n$ chẵn và bằng $(\frac{n-1}{2}+1)(\frac{n-1}{2})$ nếu $n$ lẻ.

Bài 6:

Cho $n$ hòn sỏi được đặt vào $n$ điểm trên 1 hình tròn. Mỗi lần ta dịch chuyển 2 hòn sỏi , mỗi hòn sỏi được dịch chuyển vào 1 trong 2 điểm mà nằm kề với hòn sỏi đó nhưng chiều dịch chuyển của 2 hòn sỏi đó ngược nhau (có 2 hướng dịch chuyển là cùng và ngược chiều kim đồng hồ). Ta muốn dịch chuyển tất cả các hòn sỏi vào 1 điểm. CMR:

a/ Ta có thể đạt được mục đích nếu $n$ lẻ

b/ Ta không thể đạt được mục đích nếu $n$ chẵn

Trường hợp n=3 thì có 1 csc nhưng theo kq của bạn thì có 2



#13
JUV

JUV

    Trung sĩ

  • Điều hành viên OLYMPIC
  • 138 Bài viết

Trường hợp n=3 thì có 1 csc nhưng theo kq của bạn thì có 2

Có 2 mà:(1;3),(1;2;3)



#14
JUV

JUV

    Trung sĩ

  • Điều hành viên OLYMPIC
  • 138 Bài viết

Mà tiện hỏi luôn là công sai có cần dương không? Nếu không thì mình sẽ cộng thêm $n$ vào kết quả



#15
Long Phi

Long Phi

    Binh nhất

  • Thành viên
  • 26 Bài viết

$1,3$ có phải csc đâu nhỉ, csc phải có ít nhất $3$ số mà


Bài viết đã được chỉnh sửa nội dung bởi bangbang1412: 27-05-2016 - 13:09


#16
canhhoang30011999

canhhoang30011999

    Thiếu úy

  • Thành viên
  • 634 Bài viết

Mình đọc cả $2$ cách thì thấy cách của JUV bị thừa mất $1$ số TH CSC chỉ có $2$ số nhưng mà mình thấy với $n$ đủ lớn thì đáp số của Long Phi phải lớn hơn của JUV không biết $2$ bạn ai có nhầm ở đâu không(mình đọc thấy đúng cả)?


Bài viết đã được chỉnh sửa nội dung bởi bangbang1412: 27-05-2016 - 13:09


#17
JUV

JUV

    Trung sĩ

  • Điều hành viên OLYMPIC
  • 138 Bài viết

mình đọc cả 2 cách thì thấy cách của JUV bị thừa mất 1 số TH CSC chỉ có 2 số nhưng mà mình thấy với n đủ lớn thì đáp số của Long Phi phải lớn hơn của JUV không biết 2 bạn ai có nhầm ở đâu không(mình đọc thấy đúng cả)?

Mình đã sửa lại rồi, mình tưởng là công sai phải dương nên xét thiếu TH, mà theo định nghĩa csc thì đâu cần phải có $3$ số :https://vi.wikipedia...iki/Cấp_số_cộng


Bài viết đã được chỉnh sửa nội dung bởi bangbang1412: 27-05-2016 - 13:10


#18
Long Phi

Long Phi

    Binh nhất

  • Thành viên
  • 26 Bài viết

à, mình bị nhầm, mình tính cả trường hợp 2 công bội chia hết cho nhau nữa, nếu thế lời giải của mình thay $i$ bằng các số nguyên tố, nhưng như vậy thì không rút gọn đẹp như JUV


Bài viết đã được chỉnh sửa nội dung bởi Long Phi: 27-05-2016 - 01:00


#19
bangbang1412

bangbang1412

    Độc cô cầu bại

  • Phó Quản lý Toán Cao cấp
  • 1667 Bài viết

Lời giải bài 6 :

Đánh số các điểm là $1,2,...,n$ , ta gọi giá trị hình tròn này là tổng các số dạng $a.b$ trong đó $a$ là vị trí số viên sỏi nằm ở vị trí thứ $b$

Dễ thấy đây là bất biến và bằng $\frac{n(n+1)}{2}$ khi mà đưa được hết vào một ô thì tức là có số $t$ mà $tn=\frac{n(n+1)}{2}<=>2t-1=n$ hay $n$ lẻ

Khi $n$ lẻ thì số các ô chứ sỏi không tăng ,nên một lúc nào đó sẽ về một .

Bài 7 :   Với số nguyên $m>3$ gọi $T_{m}$ là số dãy $(a_{1},a_{2},...a_{m})$ thỏa mãn

$1)$ Với mọi $i=1,2,...,m$ thì $a_{i} \in (1,2,3,4)$

$2)$ $a_{1}=a_{m}=1<a_{2}$

$3)$ Với mọi $i=3,4...,m$ thì $(a_{i-1}-a_{i})(a_{i-2}-a_{i})$ khác $0$

Tìm công thức tổng quát của dãy $(T_{n})$ và chứng minh tồn tại dãy $(g_{n})>0$ nguyên thỏa mãn $|T_{n}-g_{n}|<2\sqrt{g_{n}}$ với mọi $n$

 


Bài viết đã được chỉnh sửa nội dung bởi bangbang1412: 27-05-2016 - 01:13

$$[\Psi_f(\mathbb{1}_{X_{\eta}}) ] = \sum_{\varnothing \neq J} (-1)^{\left|J \right|-1} [\mathrm{M}_{X_{\sigma},c}^{\vee}(\widetilde{D}_J^{\circ} \times_k \mathbf{G}_{m,k}^{\left|J \right|-1})] \in K_0(\mathbf{SH}_{\mathfrak{M},ct}(X_{\sigma})).$$


#20
canhhoang30011999

canhhoang30011999

    Thiếu úy

  • Thành viên
  • 634 Bài viết

Lời giải bài 7 : 


Ta có $T_{4}=T_{5}=T_{6}=6$

Xét với $n>6$ ta có số bộ $(1,a_{2},...,a_{n},1)$ thỏa mãn đk $1,2,3$ bằng số bộ $(1,a_{2},....,a_{n-2},a_{n-1},1)$ với $a_{n-2}$ khác 1 thỏa 1,2,3 cộng với số bộ

$(1,a_{2},....,1,a_{n-1},a{n},1)$ thỏa đk $1,2,3$

nên $T_{n+1}=T{n}+4T_{n-2}$ với $n\geq 6$ (công thức tổng quát phải dùng số phức lằng nhằng nên mình không post)

còn ý b thì chọn $g_{n}=T_{n}$ là được mà

Bài 8 : (Macedonia TST $2016$)(bài này bữa post rồi mà chưa thấy ai giải nên mình cho vào đây không biết có phạm quy không)

. Cho lưới vuông kích thước 2n×2n, chứa các ô vuông đơn vị trắng. Trong một nước đi một người có thể đổi màu của ba ô liên tiếp trong cùng một hàng hoặc một cột, với quy ước trắng thành đen và đen thành trắng. Xác định tất cả số nguyên dương $n\geq 2$, sao cho với hữu hạn nước đi, ta có thể thu được một bàn cờ vua.


Bài viết đã được chỉnh sửa nội dung bởi bangbang1412: 27-05-2016 - 13:13






Được gắn nhãn với một hoặc nhiều trong số những từ khóa sau: tổ hợp rời rạc, marathon

2 người đang xem chủ đề

0 thành viên, 2 khách, 0 thành viên ẩn danh